A local hamburger shop sold a combined total of 632 hamburgers and cheeseburgers on Saturday. There were 68 fewer cheeseburgers sold than hamburgers. How many hamburgers were sold on Saturday?

Answers

Answer 1

The number of hamburger is 350 and the number of cheeseburger is 282.

How to illustrate the information?

It should be noted that an expression is simply used to show the relationship between the variables.

In this case, the local hamburger shop sold a combined total of 632 hamburgers and cheeseburgers on Saturday and there were 68 fewer cheeseburgers sold than hamburgers.

Let Cheeseburger = x

Hamburger = x + 68

This will be:

x + x + 68 = 632

2x + 68 = 632

2x = 632 - 68

2x = 564

x = 564/2

x = 282

Cheeseburger = 282

Hamburger = x + 68 = 282 + 68 = 350

Learn more about expressions on:

brainly.com/question/723406

#SPJ1


Related Questions

Find the derivative of each function. Simplify each derivative and express all exponents as positive values.

Answers

Answer:

[tex]\boxed{f^{\prime}(x)=x^2^{}-\frac{1}{2}}[/tex]

Explanation:

Step 1. The function we have is:

[tex]f(x)=\frac{x^3}{3}-\frac{x}{2}[/tex]

And we are asked to find the derivative of the function. The rule to find the derivative for this type of function is:

[tex]\begin{gathered} \text{for a function }of\text{ the form} \\ f(x)=ax^n \\ \text{The derivative is:} \\ f^{\prime}(x)=a(n)x^{n-1} \end{gathered}[/tex]

Step 2. Before we apply the derivative rule, remember the following:

[tex]\begin{gathered} \text{for a function } \\ f(x)=g(x)+h(x) \\ \text{The derivative is:} \\ f^{\prime}(x)=g^{\prime}(x)+h^{\prime}(x) \end{gathered}[/tex]

This means that we need to derivate each part or term of the function and combine them for the total derivative.

Step 3. Apply the derivative rule from step 1 to the given function.

First we rewrite the function as follows:

[tex]\begin{gathered} f(x)=\frac{x^3}{3}-\frac{x}{2} \\ \downarrow \\ f(x)=\frac{1}{3}x^3-\frac{1}{2}x^1 \end{gathered}[/tex]

Apply the derivative rule:

[tex]f^{\prime}(x)=\frac{1}{3}(3)x^{3-1}-\frac{1}{2}(1)x^{1-1}[/tex]

Step 4. The last step is to simplify the expression:

[tex]\begin{gathered} f^{\prime}(x)=\frac{1}{3}(3)x^{3-1}-\frac{1}{2}(1)x^{1-1} \\ \downarrow \\ f^{\prime}(x)=\frac{1}{3}(3)x^2-\frac{1}{2}(1)x^0 \\ f^{\prime}(x)=x^2-\frac{1}{2}x^{^0} \\ \sin ce^{} \\ x^0=1 \\ \downarrow\text{ The result is }\downarrow \\ f^{\prime}(x)=x^2-\frac{1}{2} \end{gathered}[/tex]

Answer:

[tex]\boxed{f^{\prime}(x)=x^2^{}-\frac{1}{2}}[/tex]

(4x + 7) + 35 = 90 solve for x

Answers

Answer:

x=12

Step-by-step explanation:

(4x + 7) + 35 = 90

(4x+7)=90-35
4x+7=55
4x=55-7
4x=48

x=12

Answer:

x=12

Step-by-step explanation:

it just is.,...........

At the Kerry Hotel Pudong in Shanghai, China, theonce swimming pool is now the world's largest ballpit. The ball pit is 82 feet long and 41 feet wide,with an average depth of 4.25 feet. Each ball in the ball pit is the same size. They each have a diameterof 4 inches. 1. What is the volume of this pool? 2. How many ball pit balls will we need to fill the pool up to the top?

Answers

Answer: We have to find the volume of the pool and the number of pit balls that can fill it.

(1) We can find the volume of the pool with the following formula:

[tex]V=L\times W\times D\rightarrow(1)[/tex]

Using the equation (1) the volume of the pool is determined as follows:

[tex]\begin{gathered} L=82ft \\ \\ W=41ft \\ \\ D=4.25ft \\ \\ \therefore\rightarrow \\ \\ \begin{equation*} V=L\times W\times D \end{equation*} \\ \\ V=(82ft)\times(41ft)\times(4.25ft)=14,288.5ft^3 \\ \\ V=14,288.5ft^3 \end{gathered}[/tex]

(2) The number of ball pits that can fill the pool is as follows:

[tex]\begin{gathered} V_b=\frac{4}{3}\pi r^3 \\ \\ \\ 4in=\frac{1}{3} \\ \\ \therefore\rightarrow \\ \\ V_b=\frac{4}{3}\pi(\frac{1}{3})^3=\frac{4}{81}\pi ft^3 \\ \\ V_b=\frac{4}{81}\pi ft^3=0.16ft^3 \end{gathered}[/tex]

Therefore the answer is:

[tex]\begin{gathered} N=\frac{V}{V_b}=\frac{14,288.5ft^3}{0.16ft^3}=89,303.125 \\ \\ N=89,303.125 \end{gathered}[/tex]

An architect is standing 250 feet from the base of a building and would like to know the height of the building. If he measures the angle of elevation to be 55°, what is the approximate height of the building? Round any intermediate calculations if needed to know less than six decimal places and round the final answer to the nearest 10th of a foot

Answers

see the figure below to better understand the problem

we have that

[tex]\begin{gathered} tan55^o=\frac{h}{250}---->\text{ by TOA} \\ \\ solve\text{ for h} \\ h=250*tan55^o \\ h=357.0\text{ ft} \end{gathered}[/tex]The answer is 357.0 feet

Write a mathematical expression for the following statement

Four times x is more than 13

Answers

Answer:

4x>13

Step-by-step explanation:

4 × X=4x

>=more than symbol

Jen draws a polygon with Vertices E (-2, 3.5), F (3,3.5) G (3,-1.5) And H (-2, "-1.5)." Is EFGH a square? Justify your answer

Answers

The polygon EFGH is a polygon.

What is polygon?

A polygon is a form of planar figure in geometry and is defined as a closed polygonal chain made up of a finite number of straight line segments. A region that is enclosed by a bounding plane, a bounding circuit, or both is referred to as a polygon. The portions of a polygonal circuit are referred to as its edges or sides.

To check the polygon EFGH is a square or not, first to find the length of each side of polygon.

If the length of each side is equal then the polygon EFGH is a square.

(Since length of each side of square is equal).

We know,

[tex]d \sqrt{(x_2-x_1)^2 + (y_2 - y_1)^2}[/tex]

where,

d = distance

[tex](x_1, x_2)[/tex] = Coordinates of the first point.

[tex](x_2, y_2)[/tex] = Coordinates of the second point.

So,

[tex]EF = \sqrt{(3-(-2))^2 + (3.5-3.5)^2} = \sqrt{25 + 0} = 5[/tex]

[tex]FG = \sqrt{(3-3)^2 + (-1.5-3.5)^2 } = \sqrt{0 + 25} = 5[/tex]

[tex]GH = \sqrt{(-2-3)^2 + (-1.5-(-1.5))^2} = \sqrt{25 + 0} = 5[/tex]

[tex]EH = \sqrt{(-2-(-2))^2 + (-1.5-3.5)^2} = \sqrt{0 + 25} = 5[/tex]

From above the length of each side is equal.

Therefore, given polygon EFGH is a square.

To know more about the polygon, click on the link

https://brainly.com/question/26583264

#SPJ13

what is 1 trillion to the thenth power? .. .. ..

[tex]\sf{}[/tex]

Answers

1 trillion to the tenth power is (10¹⁴)¹⁰ or  (10¹³)¹⁰.

What is a trillion?1,000,000,000,000One trillion is equal to one million million, or 1,000,000,000,000, and on the short scale, we write this as 1012. (ten to the twelfth power). This number is now commonly referred to as one trillion because it is a thousand times bigger than the short-scale billion.We need to determine how many crores one trillion represents. Therefore, by dividing one trillion by one crore, we can find the solution to the puzzle. As a result, we discover that 100000  = 1000000000000/10000000. As a result, we learn that $1 trillion is equivalent to 100,000 Indian Rupees.

So, 1 trillion to the tenth power:

1 trillion to the tenth power = (10¹⁴)¹⁰ or  (10¹³)¹⁰

Therefore, 1 trillion to the tenth power is (10¹⁴)¹⁰ or  (10¹³)¹⁰.

Know more about trillions here:

https://brainly.com/question/15016664

#SPJ3

I need help solving this question

Answers

Answer:  0.77

Step-by-step explanation:

the mean absolute
for the following set of Determine
deviation
data: 1, 1, 3, 5, 5, 6, 8, 11

Answers

The mean absolute deviation is 2.5.

What is Mean Absolute Deviation?

The average distance between each data value and the mean is known as the mean absolute deviation (MAD) of a data collection. A measure of variance in a data collection is the mean absolute deviation. We may determine how "spread out" the values in a data collection are by looking at the mean absolute deviation.

How to calculate the mean deviation from the mean:

Calculate the mean of the provided observations.Determine how much each observation differs from the estimated mean.Analyze the mean of the differences discovered in step two.

Given data:1, 1, 3, 5, 5, 6, 8, 11

So, mean of the data is

= 1+ 1 +3 +5 +5+ 6+ 8 +11 / 8

= 40 /8=

5

Now, Mean absolute deviation is

= | 1-5| + |1-5| + |3-5 | + |5-5| + |5-5| + |6-5| + |8-5| + |11-5|/ 8

= 4 + 4 + 2+ 0 + 0+ 1 + 3 +6 /8

= 20/8

=2.5

Hence, the mean absolute deviation is is 2.5.

Learn more about mean absolute deviation here:

https://brainly.com/question/23556021

#SPJ1

select the correct answer what is the simplified form of 45

Answers

[tex]\sqrt[]{45}\text{ = }\sqrt[]{9\times5}\text{ = }\sqrt[]{9}\text{ }\times\text{ }\sqrt[]{5}\text{ = 3 }\sqrt[]{5}[/tex]

The solution is option C.

Write in point slope form the equation for the line that goes through the points (0,0) and (-4,7)

Answers

PROBLEM:

To find the point-slope form of the equation of the line passing through points (0, 0) and (-4, 7)

METHOD:

The point-slope form of the equation of a line is given to be:

[tex]\begin{gathered} (y-y_0)=m(x-x_0) \\ \text{where} \\ m=\text{ slope} \\ (x_0,y_0)=\text{ Point on the line} \end{gathered}[/tex]

Step 1: Find the slope of the line.

The formula to calculate the slope of a line is given to be:

[tex]m=\frac{y_2-y_1}{x_2-x_1}[/tex]

We can use the two points provided to find the slope of the line such that:

[tex]\begin{gathered} (x_1,y_1)=(0,0)_{} \\ (x_2,y_2)=(-4,7) \end{gathered}[/tex]

Therefore, the slope is given to be:

[tex]\begin{gathered} m=\frac{7-0}{-4-0} \\ m=-\frac{7}{4} \end{gathered}[/tex]

Step 2: Pick a point on the line to use for the equation.

[tex](x_0,y_0)=(-4,7)[/tex]

Step 3: Use the values gotten from Steps 1 and 2 to write out the equation of the line in the point-slope form:

[tex]\begin{gathered} \Rightarrow y-7=-\frac{7}{4}(x-\lbrack-4\rbrack) \\ \therefore \\ y-7=-\frac{7}{4}(x+4) \end{gathered}[/tex]

ANSWER:

The slope-intercept form of the line is given to be:

[tex]y-7=-\frac{7}{4}(x+4)[/tex]

Onur is participating in a walkathon fundraiser. Two donors promised to donate money based on the total distance he walks. The money, AAA, in dollars, that he receives from the first donor given that he walks ddd kilometers is given by the formula A(d)=10dA(d)=10dA, left parenthesis, d, right parenthesis, equals, 10, d. The money, BBB, in dollars, that he receives from the second donor given that he walks ddd kilometers is given by the formula B(d)=2d+d^2B(d)=2d+d
2
B, left parenthesis, d, right parenthesis, equals, 2, d, plus, d, squared.
Let TTT be the total money that Onur raises from those donors by walking ddd kilometers in the walkathon.
Write a formula for T(d)T(d)T, left parenthesis, d, right parenthesis in terms of A(d)A(d)A, left parenthesis, d, right parenthesis and B(d)B(d)B, left parenthesis, d, right parenthesis.

Answers

The function of the total money raised from both donors after walking d kilometers is T(d) = 12d + d^2

How to determine the composite function of the total amount?

From the question, the given parameters are:

First donor at d kilometers, A(d)=10dSecond donor at d kilometers, B(d) = 2d + d^2

Also, from the question, T is the total money raised from both donors at d kilometers

This means that the equation of the total money can be represented a

T(d) = A(d) + B(d)

Substitute the known values in the above equation

So, we have the following equation

So, we have

T(d) = 10d + 2d + d^2

Evaluate the like terms in the above equation

T(d) = 12d + d^2

The equation cannot be further simplified

Hence, the composite function is T(d) = 12d + d^2

Read more about composite function at

https://brainly.com/question/10687170

#SPJ1

Complete question

Onur is participating in a walkathon fundraiser. Two donors promised to donate money based on the total distance he walks. The money, A, in dollars, that he receives from the first donor given that he walks d kilometers is given by the formula A(d)=10d

The money, B, in dollars, that he receives from the second donor given that he walks d kilometers is given by the formula B(d)=2d+d^2

Let T be the total money that Onur raises from those donors by walking d kilometers in the walkathon.

Write a formula for T(d) in terms of A(d) and B(d)

Sam biked 2/3 km in 4 minutes. How far did he bike in 1 minute?

Answers

we can make a relation

[tex]\begin{gathered} \frac{2}{3}\longrightarrow4 \\ \\ x\longrightarrow1 \end{gathered}[/tex]

where x is the missing value

if she bikes 2/3km in 4 minutes how much kilometers she bike on 1 minute?

we solv using cross multiplication

we multiply 1 and 2/3 and the divide by 4

[tex]\begin{gathered} x=\frac{1\times\frac{2}{3}}{4} \\ \\ x=\frac{\frac{2}{3}}{4} \\ \\ x=\frac{1}{6} \end{gathered}[/tex]

she will bike 1/6km in one minute

A map state where your friend lives have a scale 1/2 inch: 10 miles.
Your friend measured the distance between her town and the state capital on the map. Her measurements were 4 1/2 inches. Based on your friend's measurement, what is the actual distance in miles between her town and the state capital?

Answers

The actual distance is 90 miles between the town and the state capital based on the scale of the map

1/2 inch on the map = 10 miles

So, 1 inch = 20 miles

Distance between the town and state capital = 4 1/2 inches

The mathematical relationship between a small unit of measurement on a map, such as an inch or centimetre, and the corresponding real-world unit of distance, such as a kilometre or a mile, is known as a map scale.

The actual distance in miles between the town and the state capital is given by:

Distance between town and state capital* Scale of the map

= 4.5*20

= 90 miles

Learn more about maps:

https://brainly.com/question/13279076

#SPJ9

Given f(x) = x3 - 5, find f-'(x).

Answers

the inverse function = f^-1(x) is option C

First let's state what we are given:

f(x) = x^3 - 5

We are to find the inverse of f(x).

To do this, first let's represent f(x) with y

y = = x^3 - 5

Then, we would interchange x and y

[tex]\begin{gathered} x=y^3\text{ - 5 (make y the subject of formula)} \\ x+5=y^3 \\ find\text{ cube root both sides} \\ \sqrt[3]{y^3\text{ }}=\sqrt[3]{(x+5)} \\ y\text{ = }\sqrt[3]{(x+5)} \end{gathered}[/tex]

From the answer we got, the inverse function = f^-1(x) is option C

please help !!!!!!!!!!!!!!!!!!!!!

Answers

5⁶ is the simplified form of the expression ( 5⁶/5³ )².

What is the simplified form of the given expression?

Given the expression in the question;

( 5⁶/5³ )²

To simplify the expression; us the law of indices.

xᵃ ÷ xᵇ = xᵃ⁻ᵇ

Hence, perform the operation inside the parenthesis

( 5⁶/5³ )²

( 5⁶ ÷  5³ )²

( 5⁶⁻³ )²

( 5³ )²

To remove the parenthesis, apply the law of indices.

( xᵃ )ᵇ = xᵃ ˣ ᵇ

( 5³ )²

5³ ˣ ²

5⁶

Therefore, the simplified form of the expression is 5⁶.

Learn more about laws of indices here: https://brainly.com/question/27432311

#SPJ1

please help me out thanks

Answers

A non-zero matrix A with A² = 0 is A = [tex]\left[\begin{array}{cc}0&1\\0&0\end{array}\right][/tex].

We need to find a 2 x 2 non- zero matrix with A² = [tex]\left[\begin{array}{cc}0&0\\0&0\end{array}\right][/tex]

A² = A x A

If A = [tex]\left[\begin{array}{cc}a&b\\c&d\end{array}\right][/tex], then A x A =  [tex]\left[\begin{array}{cc}a&b\\c&d\end{array}\right][/tex] x  [tex]\left[\begin{array}{cc}a&b\\c&d\end{array}\right][/tex]

                                          = [tex]\left[\begin{array}{cc}a^2+bc&ab+bd\\ac+dc&bc+d^2\end{array}\right][/tex]

This is called matrix multiplication.

Now consider the matrixes of the form A = [tex]\left[\begin{array}{cc}0&x\\0&0\end{array}\right][/tex]

Then A² =   [tex]\left[\begin{array}{cc}0&x\\0&0\end{array}\right][/tex]  x  [tex]\left[\begin{array}{cc}0&x\\0&0\end{array}\right][/tex]

              =  [tex]\left[\begin{array}{cc}0&0\\0&0\end{array}\right][/tex]

Also Consider a matrix A = [tex]\left[\begin{array}{cc}0&0\\x&0\end{array}\right][/tex]

Then A² =    [tex]\left[\begin{array}{cc}0&0\\x&0\end{array}\right][/tex] x  [tex]\left[\begin{array}{cc}0&0\\x&0\end{array}\right][/tex]

              =  [tex]\left[\begin{array}{cc}0&0\\0&0\end{array}\right][/tex]

So any matrix of the form  [tex]\left[\begin{array}{cc}0&x\\0&0\end{array}\right][/tex] and   [tex]\left[\begin{array}{cc}0&0\\x&0\end{array}\right][/tex] with x any number, will give A² = 0

In particular, A =  [tex]\left[\begin{array}{cc}0&1\\0&0\end{array}\right][/tex] is a non-zero matrix with A² = 0.

Learn more about matrix multiplication at https://brainly.com/question/12073691

#SPJ1


What is the y-value when the x-value is 18?

Answers

Answer:   -11

Step-by-step explanation:

You may write negative infinity, positive infinity or all reals if you must use these as part of your answer. Separate numbers using commas and use the word bone if needed. Round numerical numbers to the hundredths.

Answers

Given

The function,

[tex]h(x)=|x+2|+4[/tex]

To find the type of function, domain, range, x-intercept, y-intercept, one minimum or maximum, intervals where the function is positive, intervals where the function is negative.

Explanation:

It is given that,

[tex]h(x)=|x+2|+4[/tex]

Since the function is related to modulus value.

Then, the type of the function is Absolute value function.

Also, Since

[tex]h(x)=|x+2|+4[/tex]

Then, the domain of the function is,

[tex](-\infty,\infty)[/tex]

Set h(x)=z,

[tex]z=|x+2|+4[/tex]

The range of the absolute function c | a x + b | + k is f(x) is greater than equal to k.

Then,

[tex]f(z)\ge4[/tex]

Therefore, the range is,

[tex][4,\infty)[/tex]

From, the graph

From the figure there is no x-intercept.

Also, the y-intercept is 6.

Now, to find the critical points

The costs of repairing iPads in UAE are normally distributed with a standard deviation 73 Dhs. If 8% of the costs exceed 279 Dhs, find the mean of the costs. Round your answer to the nearest diham.

Answers

The costs of repairing iPads in UAE are normally distributed with a standard deviation of 73 Dhs. If 8% of the costs exceed 279 Dhs, then the mean of the costs is 1.405072

The mean of the Costs stands for the typical sum of money spent on a product's production. Depending on how many units are made, this amount may change.

Let X be the price of fixing iPads in the UAE (in Dhs)

X ~ N (μ, σ = 73)

if 8% of the costs are greater than 297 Dhs.

P (X > 297) = 8%

P(X > 297) = 0.08

We must determine the mean value (μ)

P (X > 297) = 0.08

P [(X - μ) / σ > (297 - μ) / 73] = 0.08

P [Z < (297 - μ) / 73] = 0.08

P [Z < (297 - μ) / 73] = 1 - 0.08

P [Z < (297 - μ) / 73] = 0.92

(297 - μ) / 73 = Z⁻¹(0.92)

(297 - μ) / 73 = 1.405072    (Using R software)

297 - μ = 73 * 1.405072

μ = 297 - (73 * 1.405072)

μ = 297 - 102.5702

μ = 194.4298

μ ≅ 194

Therefore, the mean of the cost (μ) = 194 dhs.

Using R Software, we can compute;

Z⁻¹ (0.92) is,

=qnorm(0.92) ` → (Command)

'1.405072 → (Output)

⇒ Z⁻¹ (0.92) = 1.405072

To know more about costs, refer to this link:

https://brainly.com/question/14328001

#SPJ1

When given a line of 3x + 6y = 8, how do you find slope intercept?

Answers

The line is given by 3x + 6y = 8

To find the slope and the y-intercept, we must rewrite the equation in the form y = ax + b, where a is the slope and b is the y-intercept.

The first step is subtract 3x from both sides of the equation:

3x + 6y - 3x = -3x + 8

6y = -3x + 8

Then, we divide both sides by 6:

6y/6 = (-3x + 8)/6

y = -x/2 + 4/3

Therefore, the slope is -1/2 and the y-intercept is 4/3

OutcomeAnswer the following. Round your answers to the nearest thousandths.Number of SpinsExplanationCheckRed(a) From Donna's results, compute the experimental probability of landing on yellow.394аYellowJ420(b) Assuming that the spinner is fair, compute the theoretical probability of landing on yellow.(c) Assuming that the spinner is fair, choose the statement below that is true.O With a large number of spins, there might be a difference between the experimental andtheoretical probabilities, but the difference i need help with this math problem

Answers

ANSWERS

(a) 0.420

(b) 0.400

(c) With a large number of spins, there might be a difference between the experimental and theoretical probabilities, but the difference should be small

EXPLANATION

(a) The experimental probability is given by,

[tex]P_{exp}=\frac{number\text{ }of\text{ }successes}{number\text{ }of\text{ }trials}[/tex]

In this case, Donna spun the dial 1000 times and 420 times it landed on yellow,

[tex]P_{exp}=\frac{420}{1000}=0.420[/tex]

Hence, the experimental probability of landing on yellow is 0.420.

(b) The theoretical probability is given by the geometry of the spinner and, if it is fair, the number of yellow sections,

[tex]P_{th}=\frac{number\text{ }of\text{ }favorable\text{ }outcomes}{total\text{ }posible\text{ }outcomes}[/tex]

In this case, the spinner is divided into 10 equally sized slices, and 4 of them are yellow,

[tex]P_{th}=\frac{4}{10}=0.400[/tex]

Hence, the theoretical probability of landing on yellow is 0.400.

(c) As we can see in parts a and b, for 1000 trials, the experimental and theoretical probabilities are almost the same - i.e. there is a small difference. This is known as the law of large numbers which states that if the number of trials of an experiment is large, the experimental probability of each event should be close to the theoretical probability.

Hence, the true statement is the first one: with a large number of spins, there might be a difference between the experimental and theoretical probabilities, but the difference should be small.

The graph is blurry but the y axis is going up by 25 and x axis is going up by 1s

Answers

[tex]\begin{gathered} \text{ The equation of the trendline for the data is:} \\ y=5x+23 \end{gathered}[/tex]

The predicted temperature of the substance after 3.5 minutes implies that x =3.5, and we are to find y.

Therefore,

[tex]\begin{gathered} y=5(3.5)+23 \\ y=17.5+23 \\ y=40.5^0C \end{gathered}[/tex]

Therefore, the predicted temperature of the substance, 3.5 minutes after the start of the experiment is 40.5 degrees Celsius

Which of the following graphs shows a parabola with a vertex of (-4,4) and solutions of (-6,0) and (-2,0)?

Answers

Equation is -(x+4)² +4.

What is parabola?

A parabola is an approximately U-shaped, mirror-symmetrical plane curve in mathematics. It corresponds to a number of seemingly unrelated mathematical descriptions, all of which can be shown to define the same curves. A parabola can be described using a point and a line. The vertex form of a quadratic equation is y = a (x h) 2 + k as opposed to the regular quadratic form, which is an x 2 + b x + c = y. In both cases, the variables that indicate whether the parabola is facing up (+ a) or down ( a) are y, the y-coordinate, x, and a.

Given Data

Solutions (-6, 0) and (-2,0)

y = a(x - (-6)) (x-(-2))

y  = a(x+6) (x+2)

Vertex (-4,4)

at x = -4 and y = 4

4 = a(-4+6)(-4+2)

4 = a (2)(-2)

a = -1

y = -1(x+6)(x+2)

y = -(x²+8x+12)

y = =(x² + 3x +4x+4²-4²+12)

y = -(x+4)² +4

Equation is -(x+4)² +4.

To learn more about parabola, visit:

https://brainly.com/question/21685473

#SPJ9

HELP PLEASE MARKING BRAINLIEST Han cell phone plan cost $200 to start then there's $50 charged each month(a)what is the total cost to use the cell phone plan for one month(b)what is the total cost for x months (c)which graph shows the cost of the cell phone plan over a period of two years using months as the units of time(d) is there a proportional relationship between time and the cost of the cell phone plan explain how you know

Answers

Answer:

• $250

,

• y=200+50x

,

• Graph A

,

• Yes

Explanation:

(a)Han cellphone's plan costs a fixed fee of $200 and an extra $50 per month.

Therefore:

[tex]\begin{gathered} \text{The total cost for one month=}200+50 \\ =\$250 \end{gathered}[/tex]

(b)If Han uses the phone for x months

Extra Cost = 50x

Therefore:

[tex]\text{The total cost for x months, y=}200+50x[/tex]

(c)Since the number of months = x; and

Total cost for x months = y

When x=24

[tex]\begin{gathered} y=200+50(24) \\ =1400 \end{gathered}[/tex]

The graph that shows the cost of the cell phone plan over a period of two years using months as the units of time is Graph A.

(d)We see that the equation y=50x+200 has a proportional constant (50).

Another approach is to say that it is a linear equation.

Therefore, there is a proportional relationship between time and the cost of the cell phone plan.

how long will it take $600 to earn $72 at the rate of 0.03 percent annual?​

Answers

It takes 400 years for $600 to earn $72 at the rate of 0.03 percent annual

What is Simple Interest?

"Simple" interest refers to the straightforward crediting of cash flows associated with some investment or deposit.

The formula for calculating Simple interest

I=PRT

Where I is the Interest

P=Principle amount

I is interest Amount

R is rate of interest per year as a percent

T is time period involved

P=600

I=72

r=0.03%

t=?

I=PRT

Plug in the values P, R and I

72=600×(0.03%)×T

72=600×0.0003×T

72=0.18×T

400=T

Hence it takes 400 years, for amount  $600 to earn $72 at the rate of 0.03 percent annual.

To learn more on Simple Interest Click:

https://brainly.com/question/25845758

#SPJ1

Write a function and use a table and graph to predict when the population will reach 20,000

Answers

The formula for exponential growth is expressed as

y = a(1 + r)^t

where

y is the population after t years

ais the initial population

r is the growth rate

t is the number of years

From the information given,

a = 350

r = 14% = 14/100 = 0.14

By substituting these values into the equation, the function to model th growth rate is

y = 350(1 + 0.14)^t

y = 350(1.14)^t

We would substitute values of t into the equation to determine corresponding values of y. We have

For t = 2, y = 350(1.14)^2 = 454.86

For t = 6, y = 350(1.14)^6 = 768.24

For t = 10, y = 350(1.14)^10 = 1297.52

We would plot these values of y on the y axis and the corresponding values of t on the x axis. The graph is shown below

The time when the population will reach 20000 would be the value of x at the point where y = 20000 on the graph. It is around y = 30.87

Thus, the population will reach 20000 in approximately 31 years

We can confirm this by solving the equation. We have

20000 = 350(1.14)^t

20000/350 = 1.14^t

57.14 = 1.14^t

Taking natural log of both sides, we have

ln 57.14 = ln1.14^t = tln1.14

t = ln 57.14/ln1.14

t = 30.87505

HURRY!!
Describe the end behavior of f(x) = −3x4 + 7x2 − x + 13.

Answers

The end behavior of the given function will be given by Down on the left down on the right.

A function may be defined as the expression in which for one value of input variable there is only one value of output variable. The input variable or the independent variable is x, and the output variable or the dependent variable is y. In the given function f(x) = -3x⁴ + 7x² - x + 13 we will use the Power and Polynomial Functions features. For final behavior of power functions of the form f(x) = axⁿ where n is represented as a non-negative integer depends on the power and the constant. Here, the leading coefficient is -3x⁴. So, when x → ∞ the value of f(x) → -∞ and when x → -∞ then the value of f(x) → -∞. So, the final answer for the behavior will be given by "Down on the left, down on the right".

Learn more about Behavior of function at:

brainly.com/question/11275875

#SPJ1

A jar contains 5 red marbles, 7 green marbles, and 6 blue marbles.

What is the probability that you draw 3 green marbles in a row if you do replace the marbles after each draw? =
What is the probability that you draw 7 blue marbles in a row if you don't replace the marbles after each draw? =

Answers

Probability of drawing 3 green marbles in a row with replacement is 0.059

The probability that you draw 7 blue marbles in a row without replacement is 0

What is the probability?

Probability is used to determine the odds in favor or against a random event happening. The probability that the random event happens lie between 0 and 1. The more likely it is that an event would happen the closer the probability value would be to 1.

Probability of drawing 3 green marbles in a row with replacement = (7/18) x (7/18) x (7/18) = 0.059

The probability that you draw 7 blue marbles in a row without replacement = (6/18) x (5/17) x (4/16) x (3/15) x (2/14) x (1/13) x 0 = 0

To learn more about probability, please check: https://brainly.com/question/13234031

#SPJ1

Calculate the perimeter for each of the following ​

Answers

a) Perimeter =  17.4 cm

b) Perimeter   =210.6 cm

How are the perimeters calculated?

a) The perimeter of an irregular polygon = Sum of all the sides of the polygon.

Converting value of all sides to centimetres.

Perimeter = 3.4 +3.2 +4.3 +3.7+2.8

                 = 17.4 cm

b) The perimeter of an irregular polygon = Sum of all the sides of the polygon.

Converting value of all sides to centimetres.

Perimeter = 29.1+25.3+30+28.6+32.6+36.5+28.5

                 =210.6 cm

What is the perimeter of a polygon?

The total length of a polygon's boundary is what is referred to as its perimeter. To put it another way, a polygon's perimeter is equal to the sum of its sides. Due to the fact that polygons are closed plane shapes, their perimeters likewise reside in a two-dimensional plane.If the lengths of the sides of a polygon are known, it is possible to determine its area and perimeter.

To learn more about perimeter of a polygon, refer:

https://brainly.com/question/24297199

#SPJ13

Other Questions
Ty ordered a storage pod in the shape of a rectangular prism to store some extra things at his house. The storage pod has a length of 16 feet and a height of 8 feet. If the volume of the storage pod is 896 cubic feet, what is the width of the storage pod? sam is unhappy with her government. she feels that she pays too much in taxes. she wants to share her opinion with her representative, so she writes a letter. which first amendment freedoms give her the right to do this? file extensions should not be changed by the user because the operating system uses them to identify the program that should be used to open the file. how can you see signs of political geography on a map Use distributive property to multiply the 3 in the expression 3(x5)=12 Coal consumption harms the environment in all the following ways EXCEPT - I NEED THIS FAST FOR NEXT HOUR PLSSSa- Mining produces new habitats.b- It causes acid rain.c- It destroys animal habitats.d- It produces sulfur dioxide when burned. find the value or measure. Assume all lines that appear to be tangent are tangent.m(angle) TUV= Where art thou, Muse, that thou forget'st so longTo speak of that which gives thee all thy might?Spend'st thou thy fury on some worthless song,Darkening thy power to lend base subjects light?Return, forgetful Muse, and straight redeem5In gentle numbers time so idly spent;Sing to the ear that doth thy lays esteemAnd gives thy pen both skill and argument.Rise, resty Muse, my love's sweet face survey,If Time have any wrinkle graven there;10If any, be a satire to decay,And make Time's spoils despised every where.Give my love fame faster than Time wastes life;So thou prevent'st his scythe and crooked knife.What is the best paraphrase of line 9?Wake up, dear sleepyhead!Fame is fleeting, and so is life.Get up and look at his lovely face.Make him famous before he gets old. write an eight sentence paragraph that fully develops the topic, the topic is for some people, entering the work force is a better choice than going to college. Does the orbiting nucleus of an atom have a positive, negative, or neutral charge 10) 18p - 6p +3p-1 When a bright light is shined on glow in the dark powder what is happening to the electrons.a.The electrons are absorbing some of the light and are become excited.b.The atoms become negatively charged ions.c.Nothing, the electrons are not affected by light.d.The electrons move to a lower energy state. 4. Jason can travl 24 miles in hour. What is his average speed in miles per hour? does micromanagement stifle creativity and growth Volumes have been written on the subject of the struggle between England and America. Men of all ranks have embarked in the controversy, from different motives, and with various designs; but all have been ineffectual, and the period of debate is closed. Arms, as the last resource, decide the contest; the appeal was the choice of the king, and the continent hath accepted the challenge.What does this paragraph suggest about Paine?Question 1 options:He is weary of the ongoing conflict between Americans and the British.He is optimistic the British will choose to compromise with Americans.He worries America cannot defend itself against British aggression. He believes America's only recourse is to declare war on the British Which symbol correctly compares these fractions? 4/5 8/9 Although total fertility rates have been declining in many less developed countries, the total population has continued to grow. This is primarily because a high percentage of the population is...O maleO femaleO oldO receiving adequate medical careO over 15 years old What is the sign of -2 to the power of 49 at the college level, faculty members tend to share information primarily via lecture and the textbook because nearly all students learn best this way. What disadvantages does Nussbaum see in using economic development as the primary metric for measuring progress in a country?